A piece of insulated wire is shaped into a figure eight as shown in the figure below. For simplicity, model the two halves of the figure eight as circles. The radius of the upper circle is 3.00 cm and that of the lower circle is 7.00 cm. The wire has a uniform resistance per unit length of 8.00 N/m. A uniform magnetic field is applied perpendicular to the plane of the two circles, in the direction shown. The magnetic field is increasing at a constant rate of 2.30 T/s. * x x_x x * x x xx * x x x * *

College Physics
11th Edition
ISBN:9781305952300
Author:Raymond A. Serway, Chris Vuille
Publisher:Raymond A. Serway, Chris Vuille
Chapter1: Units, Trigonometry. And Vectors
Section: Chapter Questions
Problem 1CQ: Estimate the order of magnitude of the length, in meters, of each of the following; (a) a mouse, (b)...
icon
Related questions
Question
100%

7

A piece of insulated wire is shaped into a figure eight as shown in the figure below. For simplicity, model the two halves of
the figure eight as circles. The radius of the upper circle is 3.00 cm and that of the lower circle is 7.00 cm. The wire has a
uniform resistance per unit length of 8.00 N/m. A uniform magnetic field is applied perpendicular to the plane of the two
circles, in the direction shown. The magnetic field is increasing at a constant rate of 2.30 T/s.
* x x x x *
(a) Find the magnitude of the induced current in the wire.
.021
Your response differs from the correct answer by more than 100%. A
(b) Find the direction of the induced current in the wire. (Select all that apply.)
O clockwise in the upper loop
O clockwise in the lower loop
counterclockwise in the upper loop
O counterclockwise in the lower loop
x x x x x
x x x x
Transcribed Image Text:A piece of insulated wire is shaped into a figure eight as shown in the figure below. For simplicity, model the two halves of the figure eight as circles. The radius of the upper circle is 3.00 cm and that of the lower circle is 7.00 cm. The wire has a uniform resistance per unit length of 8.00 N/m. A uniform magnetic field is applied perpendicular to the plane of the two circles, in the direction shown. The magnetic field is increasing at a constant rate of 2.30 T/s. * x x x x * (a) Find the magnitude of the induced current in the wire. .021 Your response differs from the correct answer by more than 100%. A (b) Find the direction of the induced current in the wire. (Select all that apply.) O clockwise in the upper loop O clockwise in the lower loop counterclockwise in the upper loop O counterclockwise in the lower loop x x x x x x x x x
Expert Solution
trending now

Trending now

This is a popular solution!

steps

Step by step

Solved in 5 steps with 4 images

Blurred answer
Similar questions
Recommended textbooks for you
College Physics
College Physics
Physics
ISBN:
9781305952300
Author:
Raymond A. Serway, Chris Vuille
Publisher:
Cengage Learning
University Physics (14th Edition)
University Physics (14th Edition)
Physics
ISBN:
9780133969290
Author:
Hugh D. Young, Roger A. Freedman
Publisher:
PEARSON
Introduction To Quantum Mechanics
Introduction To Quantum Mechanics
Physics
ISBN:
9781107189638
Author:
Griffiths, David J., Schroeter, Darrell F.
Publisher:
Cambridge University Press
Physics for Scientists and Engineers
Physics for Scientists and Engineers
Physics
ISBN:
9781337553278
Author:
Raymond A. Serway, John W. Jewett
Publisher:
Cengage Learning
Lecture- Tutorials for Introductory Astronomy
Lecture- Tutorials for Introductory Astronomy
Physics
ISBN:
9780321820464
Author:
Edward E. Prather, Tim P. Slater, Jeff P. Adams, Gina Brissenden
Publisher:
Addison-Wesley
College Physics: A Strategic Approach (4th Editio…
College Physics: A Strategic Approach (4th Editio…
Physics
ISBN:
9780134609034
Author:
Randall D. Knight (Professor Emeritus), Brian Jones, Stuart Field
Publisher:
PEARSON